Electrodinámica clásica de acción mínima sin potenciales

¿Es posible formular la electrodinámica clásica (en el sentido de derivar las ecuaciones de Maxwell) a partir de un principio de acción mínima, sin el uso de potenciales? Es decir, ¿existe un lagrangiano que dependa únicamente de los campos eléctrico y magnético y que tenga las ecuaciones de Maxwell como sus ecuaciones de Euler-Lagrange?

Realmente no puedo ver la conexión entre la derivación de Maxwell Eq. desde el principio de mínima acción y para no usar potenciales, podemos incluir o quitar potenciales en cualquier punto, porque en física clásica (no cuántica) esto es solo una cuestión de definiciones. Con respecto a la derivación de acción mínima, para usarla necesitamos tener algo de Lagrangiano para comenzar, si supones que se da, entonces, por supuesto, puedes derivar todas las ecuaciones de Maxwell. De todos modos, una cosa interesante que sé es que si supusieras que tenemos alguna fuerza que se comporta como la gravedad (central y proporcional a las distancias cuadradas
Al revisar las respuestas, la navaja de Occam me dice que este no es el camino a seguir.

Respuestas (4)

1) Bueno, en el nivel clásico, si se nos permite introducir variables auxiliares, siempre podemos codificar trivialmente un conjunto de ecuaciones de movimiento

(1) mi o metro i = 0 , i { 1 , , norte } ,
en forma de fuerza bruta con la ayuda de los multiplicadores de Lagrange
(2) λ i , i { 1 , , norte } ,
de modo que la densidad lagrangiana simplemente lee
(3) L   =   i = 1 norte λ i   mi o metro i .

Esto, por muchas razones, no es una solución muy satisfactoria. (Especialmente si comenzamos a pensar en los aspectos de la mecánica cuántica. Sin embargo, OP solo pregunta sobre la física clásica ). Sin embargo, las reescrituras triviales anteriores (3) ilustran lo difícil que es formular y probar teoremas imposibles con argumentos herméticos.

2) Para proceder, debemos imponer condiciones adicionales a la forma del principio de acción. En primer lugar, dado que tenemos prohibido introducir potenciales de calibre A m como variables fundamentales (que podemos variar en el principio de acción), supondremos que las variables fundamentales EM en el vacío deben estar dadas por el mi y B campo. Ya en EM puro, es imposible conseguir el 1 + 1 + 3 + 3 = 8 ecuaciones de Maxwell (en forma diferencial) como ecuaciones de Euler-Lagrange. variando solo el 3 + 3 = 6 variables de campo mi y B . Así que eventualmente tendríamos que introducir variables de campo adicionales, de una forma u otra.

3a) No mejora nada si tratamos de acoplar EM a la materia. Al desacoplar rincones de la teoría, deberíamos poder recuperar casos especiales bien conocidos. Por ejemplo, en el caso de EM acoplada a partículas puntuales cargadas, digamos en un límite no relativista donde no hay campo EM, el Lagrangiano de una sola carga puntual debería reducirse a la forma bien conocida

(4) L   =   1 2 metro v 2

de una partícula libre. Una discusión de la ec. (4) se puede encontrar, por ejemplo, en esta publicación de Phys.SE. Aquí supondremos que la ec. (4) es válido en lo que sigue.

3b) La siguiente pregunta es qué sucede en la electrostática.

(5) metro v ˙   =   q mi ?

La respuesta es conocida

(6) L   =   1 2 metro v 2 V

con energía potencial

(7) V   =   q ϕ ,

donde ϕ es el potencial eléctrico escalar. Sin embargo, dado que tenemos prohibido introducir el potencial ϕ como variable fundamental, debemos interpretarla

(8) ϕ ( r )   :=   r d r mi ( r )

como funcional del campo eléctrico mi , que a su vez se toma como campo fundamental. Tenga en cuenta que las ecs. (6)-(8) corresponden a una acción no local.

3c) La generalización directa (desde la mecánica de puntos hasta la teoría de campos) de la ec. (7) es una densidad potencial

(9) V   =   ρ ϕ ,

donde ρ es una densidad de carga eléctrica. Los lectores familiarizados con el principio de acción habitual de las ecuaciones de Maxwell. reconocerá que estamos muy cerca de argumentar que el término de interacción entre EM puro y materia debe ser de la forma

(10) L i norte t   =   j m A m ,

incluso si aún no hemos discutido qué debería reemplazar el Lagrangiano estándar

(11) L mi METRO   =   1 4 F m v F m v

para EM pura.

3d) Siguiendo con la electrostática, reflexionemos sobre nuestras perspectivas de derivar la ley de Gauss en forma diferencial

(12) mi   =   ρ .

Obviamente, el rhs. de la sola ec. (12) debería aparecer variando la densidad potencial (9) wrt. uno de los tres mi campos, pero ¿cuál? El conteo no es correcto. Y porque la ec. (9) no es local, en cualquier caso obtendremos una versión integrada de ρ en vez de ρ mismo, que aparece en la derecha. de la ec. (12), y que nos propusimos reproducir.

3e) En conclusión, parece inútil acoplar una teoría EM (con mi y B como variables fundamentales) a la materia, y reproducir ecuaciones clásicas estándar. de movimiento

4) El remedio estándar es introducir 4 (definidos globalmente) potenciales de calibre A m como variables fundamentales. Esto hace 1 + 3 = 4 ecuaciones de Maxwell sin fuente. trivial, y el resto 1 + 3 = 4 ecuaciones de Maxwell con las fuentes se puede derivar variando el wrt. la 4 variables fundamentales A m .

Por ejemplo, la acción estándar (relativista especial) para EM acoplada a norte cargas puntuales masivas q 1 , , q norte , en posiciones r 1 , , r norte , se da como

(13) S [ A m ; r i ]   =   d t   L ,

donde esta el lagrangiano

(14) L   =   1 4 d 3 r   F m v F m v i = 1 norte ( metro 0 i C 2 γ ( v i ) + q i { ϕ ( r i ) v i A ( r i ) } ) .

Las ecuaciones de Euler-Lagrange correspondientes. están 4 ecuaciones de Maxwell con fuentes (al variar A m ) , y norte (relativista especial) Segundas leyes de Newton con fuerzas de Lorentz (al variar r i ) .

¡Gracias @Qmecánico! ¿Significa que incluso dadas las ecuaciones de movimiento (como las ecuaciones de Maxwell para la dinámica de campo EM y las ecuaciones de fuerza de Lorentz para la dinámica de carga) no podemos reproducir el Lagrangiano a partir de ecuaciones de movimiento? Por ejemplo, si permitimos cargas magnéticas distintas de cero, entonces no podemos usar potenciales, pero aún tenemos válidas las ecuaciones de fuerza de Maxwell y Lorentz (por supuesto, ajustadas a la existencia de cargas magnéticas). ¿Podemos formular el principio de acción mínima en este caso?
Actualicé la respuesta. La pregunta sobre los principios de acción de los monopolos magnéticos es un tema aparte muy amplio y debe publicarse como una pregunta aparte. Además de la otra pregunta Phys.SE de OP, también se ha preguntado un tema relacionado de potenciales en presencia de monopolos magnéticos en esta publicación Phys.SE y sus enlaces.

No sé si es posible otro enfoque, pero este no funciona, comenzamos con tensor F m v :

F m v = m A v v A m

pero olvídate del potencial 4 y defínelo como:

F m v = [ 0 mi X / C mi y / C mi z / C mi X / C 0 B z B y mi y / C B z 0 B X mi z / C B y B X 0 ]

y escriba la densidad lagrangiana en función de las componentes cartesianas de los campos, digamos:

L = L ( mi X , . . , B z )

y

L = 1 m 0 F m v F m v

Luego, las ecuaciones de Euler Lagrange te dan (por ejemplo, aplicadas a mi X ) L mi X = 0 entonces esto no es consistente.

¿Cómo podemos resolver el problema? Pensar en otra densidad lagrangiana, definiendo una nueva F m v tensor, eligiendo con más cuidado los campos independientes?

Se sabe que el lagrangiano EM contiene un término adicional A m j v .

¿Por qué no intentarlo? por ejemplo con un Lagrangiano de la forma específica "polinomio de segundo orden"

(1) L ( X m , X , m X ) := i , j = 1 6 ( A i j X i X j + m = 0 3 B i j m m X i X j + m , v = 0 3 C i j m v m X i v X j )
en X = ( mi X , mi y , mi z , B X , B y , B z ) . (Desde los campos clásicos X i X j = X j X i conmutar, sólo la parte simétrica de A y C juegan un papel, por lo que podemos suponer que son simétricos.) Teniendo en cuenta la respuesta de Qmechanics, no intentaremos recuperar todas las ecuaciones de Maxwell, pero intentemos, por ejemplo, las que tienen posibles "términos fuente":
(2) { mi = 0 B m = ϵ 0 mi t
Con las notaciones de ( 1 ), esto corresponde a la desaparición de 4 formas lineales (1 para la primera ecuación "escalar", 3 para la segunda ecuación vectorial; X i i-ésima componente, no algo a la potencia i )
(3) { α ( m X ) := i = 1 3 m = 0 3 α i m m X i = i = 1 3 m = 0 3 d i m m X i = i = 1 3 i X i β ( m X ) := i = 1 6 m = 0 3 β i m m X i = 1 m ( y B z z B y ) ϵ 0 mi X t = 1 m ( y X 6 z X 5 ) ϵ 0 X 1 t =
Lado izquierdo de la ecuación de Euler-Lagrange en la forma L X i = m L ( m X i ) lee
L X i = j = 1 6 ( 2 A i j X j + m = 0 3 B j i m m X j )
y rhs (con la convención de suma de Einstein)
m L ( m X i ) = m [ j = 1 6 ( m = 0 3 B i j m X j + 2 m , v = 0 3 C i j m v v X j ) ] = j = 1 6 ( m = 0 3 B i j m m X j + 2 m , v = 0 3 C i j m v m v X j )
Uno ve que para obtener ( 3 ), el lagrangiano no debe tener términos en A i j y C i j m v . Después
L X i m L ( m X i ) = j = 1 6 m = 0 3 ( B j i m B i j m ) m X j
(Precaución: B i j m notación de ( 1 ), no el campo magnético...). Solo su parte antisimétrica juega un papel, así que eligámoslo antisimétrico para tener unicidad. A mano, pongamos para i = 4 ,   0 m 3
(4) B 4 j m = B j 4 m = d j m si   1 j 3 y B 4 j m = B j 4 m = 0 si   4 j 6
(es decir, muy explícitamente la siguiente contribución   mi B X B X ( mi ) al lagrangiano ( 1 ))

Esto produce la primera ecuación de ( 3 ) ( 1 ). Para i = 2 ,   0 m 3 , intentemos

(5) B 26 2 = B 62 2 = 1 2 m   , B 25 3 = B 52 3 = 1 2 m   , B 21 0 = B 12 0 = ϵ 0 2
y B 2 j m = B j 2 m = 0 para todos los demás ( i = 2 , m , j ) diferente de los anteriores.

(Contribución explícita al Lagrangiano:

1 2 m ( y mi y B z y B z mi y ) + 1 2 m ( z mi y B y z B y mi y ) + ϵ 0 2 ( t mi y mi X t mi X mi y )
)

Conclusión parcial: en este punto no parece haber contradicción. Por cada fijo m ,   ( B i j m ) es un 6 × 6 matriz antisimétrica, por lo que 5 coeficientes independientes y 20 considerándolo todo. Imponente para recuperar ( 2 - 3 ) debería reducir ingenuamente las posibilidades a un espacio vectorial de dimensión 20 4 = dieciséis pero los ejemplos ( 4 - 5 ) muestran que uno probablemente debería pensarlo dos veces.

Estoy bastante seguro de haber leído en Arxiv un documento donde ya se había hecho algo como esto. Desafortunadamente, olvidé el nombre y no pude encontrarlo con una búsqueda rápida.
Ok, lo buscaré si tengo tiempo y reemplazaré este comentario por un enlace a ese artículo.

En la electrodinámica clásica, las cantidades físicas de interés son los campos. La teoría ya está formulada "sin" potenciales si piensas en las ecuaciones de Maxwell.

Los potenciales entran en juego más adelante si desea simplificar las ecuaciones y encontrar soluciones usando, por ejemplo, las funciones de Green, etc. Sin embargo, en la electrodinámica cuántica, los potenciales adquieren un papel físico real, véase, por ejemplo, el efecto Aharanov-Bohm.

¿Cómo aborda esto "... pero usando [un] principio de acción mínima?"
Gracias, @RobertFilter. Sin embargo, creo que se exagera la importancia del efecto Aharonov-Bohm. Todas las especulaciones sobre el papel físico de los potenciales solo son válidas si las partículas se consideran puntuales.
El acoplamiento a la materia implica explícitamente los potenciales.
@JerrySchirmer Esto es cierto, si escribimos el término de interacción en el Lagrangiano como de costumbre m A m . Sin embargo, si vuelves a las ecuaciones de Maxwell y la fuerza de Lorentz, j v F m v , los potenciales no están incluidos en ninguna ecuación de movimiento en CED. Por cierto: ¿Quién hizo de mi comentario una respuesta?
@RobertFilter: las ecuaciones de Maxwell y la fuerza de Lorentz no son parte del principio de acción. El lagrangiano es 1 4 F a b F a b + A m j m . La mayoría de los lagrangianos de materia romperán la invariancia de calibre, por lo que esto no es trivial.
@JerrySchirmer Sí, tiene razón, tuve un error tipográfico, mi término de interacción, por supuesto, debería ser j m A m como tu dices Sin embargo, las ecuaciones de movimiento de CED son ecuaciones de Maxwell y en ellas no hay potenciales, que es todo lo que digo. De todos modos, las interacciones de la materia real son, por supuesto, difíciles de manejar en CED, como podemos ver, por ejemplo, en la engorrosa derivación de la reacción de radiación.